PT1.S3.Q07 - A population of game ducks...

winter.0winter.0 Alum Member
edited July 2023 in Logical Reasoning 5 karma

Why is the correct answer A?

Admin Note: Edited title. Please use the format: "PT#.S#.Q# - brief description of question”

Comments

  • maco4538maco4538 Alum Member
    323 karma

    For any inference questions the right answer must be true whereas the wrong answers range from could be true, could be false and must be false. So here are the facts:

    • the western lake has a male:female ration of 55:45 whereas the eastern lake has a ration of 65:35.
    • the greater the disparity in gender ratios, the greater percentage of older male ducks in the population. (The greater the disparity the further from equilibrium)

    When we take these two facts into conjunctions, we get this outcome:

    The eastern lake has a greater disparity in gender ratio compared to the west and therefore has a greater percentage of older male ducks in the population compared to the west. Equally true is that the western lake has a lesser disparity in gender ration compared to the east and therefore has a lesser percentage of older make ducks compared to the east.

Sign In or Register to comment.